What is the matrix for the operator that implements a function to tell the parity of its argument?

The name of the pictureThe name of the pictureThe name of the pictureClash Royale CLAN TAG#URR8PPP











up vote
2
down vote

favorite
1












$newcommandqr[1]$ I gave myself the task of building an operator that implements the following function: $f(0) = 0$, $f(1) = 1$, $f(2) = 1$, $f(3) = 0$. I restricted myself to $x$ up to 2 bits. That is, $f$ tells the parity of its argument.



I read that in order to be reversible, a circuit $U_f$ could be defined to have the following effect: $U_fqrx_2 qr0_1 to qrx_2qr0 oplus f(x)_1$. After doing my calculations, I think the following matrix represents $U_f$



$$beginbmatrix
1 & 0 & 0 & 0\
0 & 1 & 1 & 0\
0 & 0 & 0 & 0\
0 & 0 & 0 & 0
endbmatrix otimes I_2$$



However, I'm really not sure. I once read that this definition isn't complete, that I should also define the behavior for the input $qrx_2qr1$. (I'm not too clear on that.)



So --- though confused ---, I went ahead and defined $$U_fqrx_2 qr1_1 to qrx_2qr1 oplus f(x)_1.$$ Doing my calculations again, I get the following matrix



$$beginbmatrix
0 & 1 & 1 & 0\
1 & 0 & 0 & 0\
0 & 0 & 0 & 0\
0 & 0 & 0 & 0
endbmatrix otimes I_2$$



Now I don't know which should be my matrix.










share|improve this question





















  • What is the context of all of this?
    – user1271772
    8 hours ago






  • 1




    Your matrices are wrong. For the first one, the last column is missing a non-zero entry. The top-right cell should be 1 because that's the 3->0 cell and f(3) = 0.
    – Craig Gidney
    8 hours ago














up vote
2
down vote

favorite
1












$newcommandqr[1]$ I gave myself the task of building an operator that implements the following function: $f(0) = 0$, $f(1) = 1$, $f(2) = 1$, $f(3) = 0$. I restricted myself to $x$ up to 2 bits. That is, $f$ tells the parity of its argument.



I read that in order to be reversible, a circuit $U_f$ could be defined to have the following effect: $U_fqrx_2 qr0_1 to qrx_2qr0 oplus f(x)_1$. After doing my calculations, I think the following matrix represents $U_f$



$$beginbmatrix
1 & 0 & 0 & 0\
0 & 1 & 1 & 0\
0 & 0 & 0 & 0\
0 & 0 & 0 & 0
endbmatrix otimes I_2$$



However, I'm really not sure. I once read that this definition isn't complete, that I should also define the behavior for the input $qrx_2qr1$. (I'm not too clear on that.)



So --- though confused ---, I went ahead and defined $$U_fqrx_2 qr1_1 to qrx_2qr1 oplus f(x)_1.$$ Doing my calculations again, I get the following matrix



$$beginbmatrix
0 & 1 & 1 & 0\
1 & 0 & 0 & 0\
0 & 0 & 0 & 0\
0 & 0 & 0 & 0
endbmatrix otimes I_2$$



Now I don't know which should be my matrix.










share|improve this question





















  • What is the context of all of this?
    – user1271772
    8 hours ago






  • 1




    Your matrices are wrong. For the first one, the last column is missing a non-zero entry. The top-right cell should be 1 because that's the 3->0 cell and f(3) = 0.
    – Craig Gidney
    8 hours ago












up vote
2
down vote

favorite
1









up vote
2
down vote

favorite
1






1





$newcommandqr[1]$ I gave myself the task of building an operator that implements the following function: $f(0) = 0$, $f(1) = 1$, $f(2) = 1$, $f(3) = 0$. I restricted myself to $x$ up to 2 bits. That is, $f$ tells the parity of its argument.



I read that in order to be reversible, a circuit $U_f$ could be defined to have the following effect: $U_fqrx_2 qr0_1 to qrx_2qr0 oplus f(x)_1$. After doing my calculations, I think the following matrix represents $U_f$



$$beginbmatrix
1 & 0 & 0 & 0\
0 & 1 & 1 & 0\
0 & 0 & 0 & 0\
0 & 0 & 0 & 0
endbmatrix otimes I_2$$



However, I'm really not sure. I once read that this definition isn't complete, that I should also define the behavior for the input $qrx_2qr1$. (I'm not too clear on that.)



So --- though confused ---, I went ahead and defined $$U_fqrx_2 qr1_1 to qrx_2qr1 oplus f(x)_1.$$ Doing my calculations again, I get the following matrix



$$beginbmatrix
0 & 1 & 1 & 0\
1 & 0 & 0 & 0\
0 & 0 & 0 & 0\
0 & 0 & 0 & 0
endbmatrix otimes I_2$$



Now I don't know which should be my matrix.










share|improve this question













$newcommandqr[1]$ I gave myself the task of building an operator that implements the following function: $f(0) = 0$, $f(1) = 1$, $f(2) = 1$, $f(3) = 0$. I restricted myself to $x$ up to 2 bits. That is, $f$ tells the parity of its argument.



I read that in order to be reversible, a circuit $U_f$ could be defined to have the following effect: $U_fqrx_2 qr0_1 to qrx_2qr0 oplus f(x)_1$. After doing my calculations, I think the following matrix represents $U_f$



$$beginbmatrix
1 & 0 & 0 & 0\
0 & 1 & 1 & 0\
0 & 0 & 0 & 0\
0 & 0 & 0 & 0
endbmatrix otimes I_2$$



However, I'm really not sure. I once read that this definition isn't complete, that I should also define the behavior for the input $qrx_2qr1$. (I'm not too clear on that.)



So --- though confused ---, I went ahead and defined $$U_fqrx_2 qr1_1 to qrx_2qr1 oplus f(x)_1.$$ Doing my calculations again, I get the following matrix



$$beginbmatrix
0 & 1 & 1 & 0\
1 & 0 & 0 & 0\
0 & 0 & 0 & 0\
0 & 0 & 0 & 0
endbmatrix otimes I_2$$



Now I don't know which should be my matrix.







quantum-gate matrix-representation






share|improve this question













share|improve this question











share|improve this question




share|improve this question










asked 9 hours ago









R. Chopin

3077




3077











  • What is the context of all of this?
    – user1271772
    8 hours ago






  • 1




    Your matrices are wrong. For the first one, the last column is missing a non-zero entry. The top-right cell should be 1 because that's the 3->0 cell and f(3) = 0.
    – Craig Gidney
    8 hours ago
















  • What is the context of all of this?
    – user1271772
    8 hours ago






  • 1




    Your matrices are wrong. For the first one, the last column is missing a non-zero entry. The top-right cell should be 1 because that's the 3->0 cell and f(3) = 0.
    – Craig Gidney
    8 hours ago















What is the context of all of this?
– user1271772
8 hours ago




What is the context of all of this?
– user1271772
8 hours ago




1




1




Your matrices are wrong. For the first one, the last column is missing a non-zero entry. The top-right cell should be 1 because that's the 3->0 cell and f(3) = 0.
– Craig Gidney
8 hours ago




Your matrices are wrong. For the first one, the last column is missing a non-zero entry. The top-right cell should be 1 because that's the 3->0 cell and f(3) = 0.
– Craig Gidney
8 hours ago










3 Answers
3






active

oldest

votes

















up vote
3
down vote













Since your desired operation is a non-injective function, you need a third qubit and a unitary acting on all three qubits. Using an operator on your two input qubits and tensoring this with $rm I_2$ on the third qubit is not going to work as you might as well forget about the third qubit completely if that were the case.



By the way, the two matrices you present aren't unitary ($rm U^daggerrm U=rm I_4$ with $^dagger$ the conjugate transpose does not hold), and tensoring a non-unitary matrix with $rm I_2$ won't make it unitary either.



So, let's start over: take three qubits, define the first two as your input registers and the third as your output register. Then, what you want your operation to do, is this:
$$rm U_f|000rangle=|000rangle\
rm U_f|010rangle=|011rangle\
rm U_f|100rangle=|101rangle\
rm U_f|110rangle=|110rangle$$

Note that to fully define $rm U_f$, we would have to determine a desired output for the four other possible (computational basis) input states, $|!*!*1rangle$. How we do this is entirely up to us! So we can just choose something convenient, such as this:
$$rm U_f|001rangle=|001rangle\
rm U_f|011rangle=|010rangle\
rm U_f|101rangle=|100rangle\
rm U_f|111rangle=|111rangle$$

This has the following matrix form:
$$rm U_f=beginbmatrix
1&0&0&0&0&0&0&0\
0&1&0&0&0&0&0&0\
0&0&0&1&0&0&0&0\
0&0&1&0&0&0&0&0\
0&0&0&0&0&1&0&0\
0&0&0&0&1&0&0&0\
0&0&0&0&0&0&1&0\
0&0&0&0&0&0&0&1
endbmatrix$$

Intuitively, this unitary is constructed with two CNOT gates, which swap the third qubit $|0rangleto|1rangle$ or $|1rangleto|0rangle$ if and only if one of the first two qubits is in a $|1rangle$ state:



enter image description here



You can check that multiplying the matrices of the two CNOTs indeed gives the same $rm U_f$.






share|improve this answer




















  • Note this manifestly generalizes if you make x , d bits instead. You just have a CNOT for each of the d. In fact if you break this circuit, you see the parity of the first k bits for one part and the last d-k for the other.
    – AHusain
    3 hours ago

















up vote
2
down vote













All quantum operators must be unitary. Unitary means the conjugate-transpose of the operator is its inverse. In your case:



$UU^dagger = beginbmatrix
1 & 0 & 0 & 0\
0 & 1 & 1 & 0\
0 & 0 & 0 & 0\
0 & 0 & 0 & 0
endbmatrix
beginbmatrix
1 & 0 & 0 & 0\
0 & 1 & 0 & 0\
0 & 1 & 0 & 0\
0 & 0 & 0 & 0
endbmatrix =
beginbmatrix
1 & 0 & 0 & 0\
0 & 2 & 0 & 0\
0 & 0 & 0 & 0\
0 & 0 & 0 & 0
endbmatrix$



So it is most certainly not unitary because $UU^dagger neq mathbbI_4$ (same as your second attempt).



There's a long way to construct functions like this, and a short way. The long way is to write out all inputs and outputs:



$U|000rangle = |000rangle$



$U|001rangle = |001rangle$



$U|010rangle = |011rangle$



$U|011rangle = |010rangle$



$U|100rangle = |101rangle$



$U|101rangle = |100rangle$



$U|110rangle = |110rangle$



$U|111rangle = |111rangle$



You can then pretty easily construct the operator from this.



An easier way is to use projection operators & matrix addition to implement "if-then" semantics with matrices:



$U = |00ranglelangle00| otimes mathbbI_2 + |01ranglelangle01|otimes X_2 + |10ranglelangle10| otimes X_2 + |11ranglelangle11|otimes mathbbI_2$



The way to read this is "if the input is $|00rangle$ or $|11rangle$, do not flip the third bit. If the input is $|01rangle$ or $|10rangle$, flip the third bit. $|phiranglelangle phi|$ is called the outer product, and is defined for example as follows:



$|0ranglelangle0| = beginbmatrix 1 \ 0 endbmatrix beginbmatrix 1 & 0 endbmatrix = beginbmatrix 1 & 0 \ 0 & 0 endbmatrix$



which is called a projection operator. Use projection operators to only apply an operation on specific states - here, $|0rangle$.



A huge benefit of this projectors & addition approach is that you never have to actually write out the full matrix, which can become enormous as the number of qbits increase - 3-qbit 8x8 matrices already have 64 elements! This is your first step into using symbolic rather than matrix reasoning. For example, we can use the rules of linear algebra to calculate the action of our $U$ on some input:



$U|101rangle = (|00ranglelangle00| otimes mathbbI_2 + |01ranglelangle01|otimes X_2 + |10ranglelangle10| otimes X_2 + |11ranglelangle11|otimes mathbbI_2)|101rangle$



Now, matrix multiplication distributes over addition. This means we have:



$|00ranglelangle00| otimes mathbbI_2 |101rangle + |01ranglelangle01|otimes X_2 |101rangle + |10ranglelangle10| otimes X_2 |101rangle + |11ranglelangle11|otimes mathbbI_2 |101rangle$



Let's apply further transformation rules. Note $|101rangle = |10rangle otimes |1rangle$, and $(Uotimes V)(|xrangle otimes |yrangle) = U|xrangle otimes V|yrangle$, where in our cases (for example) $U = |00ranglelangle00|$, $V = mathbbI_2$, $x=|10rangle$ and $y=|1rangle$:



$|00ranglelangle00|10rangle otimes mathbbI_2 |1rangle + |01ranglelangle01|10rangle otimes X_2 |1rangle + |10ranglelangle10|10rangle otimes X_2 |1rangle + |11ranglelangle11|10rangle otimes mathbbI_2 |1rangle$



Now, note we have the following four terms:



$langle00|10rangle, langle01|10rangle, langle10|10rangle, langle11|10rangle$



These are called inner products, or dot products and here all of them are zero except for $langle10|10rangle$ - the dot product of $|10rangle$ with itself:



$langle10|10rangle = beginbmatrix 0 & 0 & 1 & 0endbmatrix
beginbmatrix0 \ 0 \ 1 \ 0endbmatrix = 1$



Since the other terms are all zero, they all cancel out:



$requirecancel cancel + cancel1rangle + |10ranglecdot 1 otimes X_2 |1rangle + cancel1rangle$



So we are left with:



$|10rangle otimes X_2 |1rangle$



Where of course $X_2|1rangle = |0rangle$, so:



$|10rangle otimes |0rangle = |100rangle$



And we calculated $U|101rangle = |100rangle$ as expected, without once having to write out a huge inconvenient matrix!






share|improve this answer





























    up vote
    0
    down vote













    Your bit strings x in the case when you specify 2 bits are 00, 01, 10, 11.
    Now you want to output the result in another bit/qubit.



    Whether the outpit bit/qubit is initialized as 0 or 1 should not change your unitary operation. The only thing changing is the initial quantum state your system is onto where you apply the unitary operation representing your function.



    In this case, the implementation is straightforward. This is a first CNOT with first qubit of register x as control and the output qubit as target followed by another CNOT with the second qubit of x as control. If you input x=00, nothing happens. If x=01 or 10, you apply one CNOT which adds 1 to the target (definition of CNOT). If x=11, the two CNOTs are applied so you add 1 to the target twice and 1 + 1 = 0 when having one output bit.



    To get the unitary, you can just take the unitary matrix of the CNOT, tensor product with the identity matrix (but respect the order when you write). You do this for the first CNOT giving you one unitary $U_1$, then the second giving $ U_2 $ and finally multiply $ U_2 * U_1 $ to get your unitary operator.






    share|improve this answer




















      Your Answer




      StackExchange.ifUsing("editor", function ()
      return StackExchange.using("mathjaxEditing", function ()
      StackExchange.MarkdownEditor.creationCallbacks.add(function (editor, postfix)
      StackExchange.mathjaxEditing.prepareWmdForMathJax(editor, postfix, [["$", "$"], ["\\(","\\)"]]);
      );
      );
      , "mathjax-editing");

      StackExchange.ready(function()
      var channelOptions =
      tags: "".split(" "),
      id: "694"
      ;
      initTagRenderer("".split(" "), "".split(" "), channelOptions);

      StackExchange.using("externalEditor", function()
      // Have to fire editor after snippets, if snippets enabled
      if (StackExchange.settings.snippets.snippetsEnabled)
      StackExchange.using("snippets", function()
      createEditor();
      );

      else
      createEditor();

      );

      function createEditor()
      StackExchange.prepareEditor(
      heartbeatType: 'answer',
      convertImagesToLinks: false,
      noModals: false,
      showLowRepImageUploadWarning: true,
      reputationToPostImages: null,
      bindNavPrevention: true,
      postfix: "",
      noCode: true, onDemand: true,
      discardSelector: ".discard-answer"
      ,immediatelyShowMarkdownHelp:true
      );



      );













       

      draft saved


      draft discarded


















      StackExchange.ready(
      function ()
      StackExchange.openid.initPostLogin('.new-post-login', 'https%3a%2f%2fquantumcomputing.stackexchange.com%2fquestions%2f4442%2fwhat-is-the-matrix-for-the-operator-that-implements-a-function-to-tell-the-parit%23new-answer', 'question_page');

      );

      Post as a guest






























      3 Answers
      3






      active

      oldest

      votes








      3 Answers
      3






      active

      oldest

      votes









      active

      oldest

      votes






      active

      oldest

      votes








      up vote
      3
      down vote













      Since your desired operation is a non-injective function, you need a third qubit and a unitary acting on all three qubits. Using an operator on your two input qubits and tensoring this with $rm I_2$ on the third qubit is not going to work as you might as well forget about the third qubit completely if that were the case.



      By the way, the two matrices you present aren't unitary ($rm U^daggerrm U=rm I_4$ with $^dagger$ the conjugate transpose does not hold), and tensoring a non-unitary matrix with $rm I_2$ won't make it unitary either.



      So, let's start over: take three qubits, define the first two as your input registers and the third as your output register. Then, what you want your operation to do, is this:
      $$rm U_f|000rangle=|000rangle\
      rm U_f|010rangle=|011rangle\
      rm U_f|100rangle=|101rangle\
      rm U_f|110rangle=|110rangle$$

      Note that to fully define $rm U_f$, we would have to determine a desired output for the four other possible (computational basis) input states, $|!*!*1rangle$. How we do this is entirely up to us! So we can just choose something convenient, such as this:
      $$rm U_f|001rangle=|001rangle\
      rm U_f|011rangle=|010rangle\
      rm U_f|101rangle=|100rangle\
      rm U_f|111rangle=|111rangle$$

      This has the following matrix form:
      $$rm U_f=beginbmatrix
      1&0&0&0&0&0&0&0\
      0&1&0&0&0&0&0&0\
      0&0&0&1&0&0&0&0\
      0&0&1&0&0&0&0&0\
      0&0&0&0&0&1&0&0\
      0&0&0&0&1&0&0&0\
      0&0&0&0&0&0&1&0\
      0&0&0&0&0&0&0&1
      endbmatrix$$

      Intuitively, this unitary is constructed with two CNOT gates, which swap the third qubit $|0rangleto|1rangle$ or $|1rangleto|0rangle$ if and only if one of the first two qubits is in a $|1rangle$ state:



      enter image description here



      You can check that multiplying the matrices of the two CNOTs indeed gives the same $rm U_f$.






      share|improve this answer




















      • Note this manifestly generalizes if you make x , d bits instead. You just have a CNOT for each of the d. In fact if you break this circuit, you see the parity of the first k bits for one part and the last d-k for the other.
        – AHusain
        3 hours ago














      up vote
      3
      down vote













      Since your desired operation is a non-injective function, you need a third qubit and a unitary acting on all three qubits. Using an operator on your two input qubits and tensoring this with $rm I_2$ on the third qubit is not going to work as you might as well forget about the third qubit completely if that were the case.



      By the way, the two matrices you present aren't unitary ($rm U^daggerrm U=rm I_4$ with $^dagger$ the conjugate transpose does not hold), and tensoring a non-unitary matrix with $rm I_2$ won't make it unitary either.



      So, let's start over: take three qubits, define the first two as your input registers and the third as your output register. Then, what you want your operation to do, is this:
      $$rm U_f|000rangle=|000rangle\
      rm U_f|010rangle=|011rangle\
      rm U_f|100rangle=|101rangle\
      rm U_f|110rangle=|110rangle$$

      Note that to fully define $rm U_f$, we would have to determine a desired output for the four other possible (computational basis) input states, $|!*!*1rangle$. How we do this is entirely up to us! So we can just choose something convenient, such as this:
      $$rm U_f|001rangle=|001rangle\
      rm U_f|011rangle=|010rangle\
      rm U_f|101rangle=|100rangle\
      rm U_f|111rangle=|111rangle$$

      This has the following matrix form:
      $$rm U_f=beginbmatrix
      1&0&0&0&0&0&0&0\
      0&1&0&0&0&0&0&0\
      0&0&0&1&0&0&0&0\
      0&0&1&0&0&0&0&0\
      0&0&0&0&0&1&0&0\
      0&0&0&0&1&0&0&0\
      0&0&0&0&0&0&1&0\
      0&0&0&0&0&0&0&1
      endbmatrix$$

      Intuitively, this unitary is constructed with two CNOT gates, which swap the third qubit $|0rangleto|1rangle$ or $|1rangleto|0rangle$ if and only if one of the first two qubits is in a $|1rangle$ state:



      enter image description here



      You can check that multiplying the matrices of the two CNOTs indeed gives the same $rm U_f$.






      share|improve this answer




















      • Note this manifestly generalizes if you make x , d bits instead. You just have a CNOT for each of the d. In fact if you break this circuit, you see the parity of the first k bits for one part and the last d-k for the other.
        – AHusain
        3 hours ago












      up vote
      3
      down vote










      up vote
      3
      down vote









      Since your desired operation is a non-injective function, you need a third qubit and a unitary acting on all three qubits. Using an operator on your two input qubits and tensoring this with $rm I_2$ on the third qubit is not going to work as you might as well forget about the third qubit completely if that were the case.



      By the way, the two matrices you present aren't unitary ($rm U^daggerrm U=rm I_4$ with $^dagger$ the conjugate transpose does not hold), and tensoring a non-unitary matrix with $rm I_2$ won't make it unitary either.



      So, let's start over: take three qubits, define the first two as your input registers and the third as your output register. Then, what you want your operation to do, is this:
      $$rm U_f|000rangle=|000rangle\
      rm U_f|010rangle=|011rangle\
      rm U_f|100rangle=|101rangle\
      rm U_f|110rangle=|110rangle$$

      Note that to fully define $rm U_f$, we would have to determine a desired output for the four other possible (computational basis) input states, $|!*!*1rangle$. How we do this is entirely up to us! So we can just choose something convenient, such as this:
      $$rm U_f|001rangle=|001rangle\
      rm U_f|011rangle=|010rangle\
      rm U_f|101rangle=|100rangle\
      rm U_f|111rangle=|111rangle$$

      This has the following matrix form:
      $$rm U_f=beginbmatrix
      1&0&0&0&0&0&0&0\
      0&1&0&0&0&0&0&0\
      0&0&0&1&0&0&0&0\
      0&0&1&0&0&0&0&0\
      0&0&0&0&0&1&0&0\
      0&0&0&0&1&0&0&0\
      0&0&0&0&0&0&1&0\
      0&0&0&0&0&0&0&1
      endbmatrix$$

      Intuitively, this unitary is constructed with two CNOT gates, which swap the third qubit $|0rangleto|1rangle$ or $|1rangleto|0rangle$ if and only if one of the first two qubits is in a $|1rangle$ state:



      enter image description here



      You can check that multiplying the matrices of the two CNOTs indeed gives the same $rm U_f$.






      share|improve this answer












      Since your desired operation is a non-injective function, you need a third qubit and a unitary acting on all three qubits. Using an operator on your two input qubits and tensoring this with $rm I_2$ on the third qubit is not going to work as you might as well forget about the third qubit completely if that were the case.



      By the way, the two matrices you present aren't unitary ($rm U^daggerrm U=rm I_4$ with $^dagger$ the conjugate transpose does not hold), and tensoring a non-unitary matrix with $rm I_2$ won't make it unitary either.



      So, let's start over: take three qubits, define the first two as your input registers and the third as your output register. Then, what you want your operation to do, is this:
      $$rm U_f|000rangle=|000rangle\
      rm U_f|010rangle=|011rangle\
      rm U_f|100rangle=|101rangle\
      rm U_f|110rangle=|110rangle$$

      Note that to fully define $rm U_f$, we would have to determine a desired output for the four other possible (computational basis) input states, $|!*!*1rangle$. How we do this is entirely up to us! So we can just choose something convenient, such as this:
      $$rm U_f|001rangle=|001rangle\
      rm U_f|011rangle=|010rangle\
      rm U_f|101rangle=|100rangle\
      rm U_f|111rangle=|111rangle$$

      This has the following matrix form:
      $$rm U_f=beginbmatrix
      1&0&0&0&0&0&0&0\
      0&1&0&0&0&0&0&0\
      0&0&0&1&0&0&0&0\
      0&0&1&0&0&0&0&0\
      0&0&0&0&0&1&0&0\
      0&0&0&0&1&0&0&0\
      0&0&0&0&0&0&1&0\
      0&0&0&0&0&0&0&1
      endbmatrix$$

      Intuitively, this unitary is constructed with two CNOT gates, which swap the third qubit $|0rangleto|1rangle$ or $|1rangleto|0rangle$ if and only if one of the first two qubits is in a $|1rangle$ state:



      enter image description here



      You can check that multiplying the matrices of the two CNOTs indeed gives the same $rm U_f$.







      share|improve this answer












      share|improve this answer



      share|improve this answer










      answered 8 hours ago









      Dyon J Don Kiwi van Vreumingen

      1826




      1826











      • Note this manifestly generalizes if you make x , d bits instead. You just have a CNOT for each of the d. In fact if you break this circuit, you see the parity of the first k bits for one part and the last d-k for the other.
        – AHusain
        3 hours ago
















      • Note this manifestly generalizes if you make x , d bits instead. You just have a CNOT for each of the d. In fact if you break this circuit, you see the parity of the first k bits for one part and the last d-k for the other.
        – AHusain
        3 hours ago















      Note this manifestly generalizes if you make x , d bits instead. You just have a CNOT for each of the d. In fact if you break this circuit, you see the parity of the first k bits for one part and the last d-k for the other.
      – AHusain
      3 hours ago




      Note this manifestly generalizes if you make x , d bits instead. You just have a CNOT for each of the d. In fact if you break this circuit, you see the parity of the first k bits for one part and the last d-k for the other.
      – AHusain
      3 hours ago












      up vote
      2
      down vote













      All quantum operators must be unitary. Unitary means the conjugate-transpose of the operator is its inverse. In your case:



      $UU^dagger = beginbmatrix
      1 & 0 & 0 & 0\
      0 & 1 & 1 & 0\
      0 & 0 & 0 & 0\
      0 & 0 & 0 & 0
      endbmatrix
      beginbmatrix
      1 & 0 & 0 & 0\
      0 & 1 & 0 & 0\
      0 & 1 & 0 & 0\
      0 & 0 & 0 & 0
      endbmatrix =
      beginbmatrix
      1 & 0 & 0 & 0\
      0 & 2 & 0 & 0\
      0 & 0 & 0 & 0\
      0 & 0 & 0 & 0
      endbmatrix$



      So it is most certainly not unitary because $UU^dagger neq mathbbI_4$ (same as your second attempt).



      There's a long way to construct functions like this, and a short way. The long way is to write out all inputs and outputs:



      $U|000rangle = |000rangle$



      $U|001rangle = |001rangle$



      $U|010rangle = |011rangle$



      $U|011rangle = |010rangle$



      $U|100rangle = |101rangle$



      $U|101rangle = |100rangle$



      $U|110rangle = |110rangle$



      $U|111rangle = |111rangle$



      You can then pretty easily construct the operator from this.



      An easier way is to use projection operators & matrix addition to implement "if-then" semantics with matrices:



      $U = |00ranglelangle00| otimes mathbbI_2 + |01ranglelangle01|otimes X_2 + |10ranglelangle10| otimes X_2 + |11ranglelangle11|otimes mathbbI_2$



      The way to read this is "if the input is $|00rangle$ or $|11rangle$, do not flip the third bit. If the input is $|01rangle$ or $|10rangle$, flip the third bit. $|phiranglelangle phi|$ is called the outer product, and is defined for example as follows:



      $|0ranglelangle0| = beginbmatrix 1 \ 0 endbmatrix beginbmatrix 1 & 0 endbmatrix = beginbmatrix 1 & 0 \ 0 & 0 endbmatrix$



      which is called a projection operator. Use projection operators to only apply an operation on specific states - here, $|0rangle$.



      A huge benefit of this projectors & addition approach is that you never have to actually write out the full matrix, which can become enormous as the number of qbits increase - 3-qbit 8x8 matrices already have 64 elements! This is your first step into using symbolic rather than matrix reasoning. For example, we can use the rules of linear algebra to calculate the action of our $U$ on some input:



      $U|101rangle = (|00ranglelangle00| otimes mathbbI_2 + |01ranglelangle01|otimes X_2 + |10ranglelangle10| otimes X_2 + |11ranglelangle11|otimes mathbbI_2)|101rangle$



      Now, matrix multiplication distributes over addition. This means we have:



      $|00ranglelangle00| otimes mathbbI_2 |101rangle + |01ranglelangle01|otimes X_2 |101rangle + |10ranglelangle10| otimes X_2 |101rangle + |11ranglelangle11|otimes mathbbI_2 |101rangle$



      Let's apply further transformation rules. Note $|101rangle = |10rangle otimes |1rangle$, and $(Uotimes V)(|xrangle otimes |yrangle) = U|xrangle otimes V|yrangle$, where in our cases (for example) $U = |00ranglelangle00|$, $V = mathbbI_2$, $x=|10rangle$ and $y=|1rangle$:



      $|00ranglelangle00|10rangle otimes mathbbI_2 |1rangle + |01ranglelangle01|10rangle otimes X_2 |1rangle + |10ranglelangle10|10rangle otimes X_2 |1rangle + |11ranglelangle11|10rangle otimes mathbbI_2 |1rangle$



      Now, note we have the following four terms:



      $langle00|10rangle, langle01|10rangle, langle10|10rangle, langle11|10rangle$



      These are called inner products, or dot products and here all of them are zero except for $langle10|10rangle$ - the dot product of $|10rangle$ with itself:



      $langle10|10rangle = beginbmatrix 0 & 0 & 1 & 0endbmatrix
      beginbmatrix0 \ 0 \ 1 \ 0endbmatrix = 1$



      Since the other terms are all zero, they all cancel out:



      $requirecancel cancel + cancel1rangle + |10ranglecdot 1 otimes X_2 |1rangle + cancel1rangle$



      So we are left with:



      $|10rangle otimes X_2 |1rangle$



      Where of course $X_2|1rangle = |0rangle$, so:



      $|10rangle otimes |0rangle = |100rangle$



      And we calculated $U|101rangle = |100rangle$ as expected, without once having to write out a huge inconvenient matrix!






      share|improve this answer


























        up vote
        2
        down vote













        All quantum operators must be unitary. Unitary means the conjugate-transpose of the operator is its inverse. In your case:



        $UU^dagger = beginbmatrix
        1 & 0 & 0 & 0\
        0 & 1 & 1 & 0\
        0 & 0 & 0 & 0\
        0 & 0 & 0 & 0
        endbmatrix
        beginbmatrix
        1 & 0 & 0 & 0\
        0 & 1 & 0 & 0\
        0 & 1 & 0 & 0\
        0 & 0 & 0 & 0
        endbmatrix =
        beginbmatrix
        1 & 0 & 0 & 0\
        0 & 2 & 0 & 0\
        0 & 0 & 0 & 0\
        0 & 0 & 0 & 0
        endbmatrix$



        So it is most certainly not unitary because $UU^dagger neq mathbbI_4$ (same as your second attempt).



        There's a long way to construct functions like this, and a short way. The long way is to write out all inputs and outputs:



        $U|000rangle = |000rangle$



        $U|001rangle = |001rangle$



        $U|010rangle = |011rangle$



        $U|011rangle = |010rangle$



        $U|100rangle = |101rangle$



        $U|101rangle = |100rangle$



        $U|110rangle = |110rangle$



        $U|111rangle = |111rangle$



        You can then pretty easily construct the operator from this.



        An easier way is to use projection operators & matrix addition to implement "if-then" semantics with matrices:



        $U = |00ranglelangle00| otimes mathbbI_2 + |01ranglelangle01|otimes X_2 + |10ranglelangle10| otimes X_2 + |11ranglelangle11|otimes mathbbI_2$



        The way to read this is "if the input is $|00rangle$ or $|11rangle$, do not flip the third bit. If the input is $|01rangle$ or $|10rangle$, flip the third bit. $|phiranglelangle phi|$ is called the outer product, and is defined for example as follows:



        $|0ranglelangle0| = beginbmatrix 1 \ 0 endbmatrix beginbmatrix 1 & 0 endbmatrix = beginbmatrix 1 & 0 \ 0 & 0 endbmatrix$



        which is called a projection operator. Use projection operators to only apply an operation on specific states - here, $|0rangle$.



        A huge benefit of this projectors & addition approach is that you never have to actually write out the full matrix, which can become enormous as the number of qbits increase - 3-qbit 8x8 matrices already have 64 elements! This is your first step into using symbolic rather than matrix reasoning. For example, we can use the rules of linear algebra to calculate the action of our $U$ on some input:



        $U|101rangle = (|00ranglelangle00| otimes mathbbI_2 + |01ranglelangle01|otimes X_2 + |10ranglelangle10| otimes X_2 + |11ranglelangle11|otimes mathbbI_2)|101rangle$



        Now, matrix multiplication distributes over addition. This means we have:



        $|00ranglelangle00| otimes mathbbI_2 |101rangle + |01ranglelangle01|otimes X_2 |101rangle + |10ranglelangle10| otimes X_2 |101rangle + |11ranglelangle11|otimes mathbbI_2 |101rangle$



        Let's apply further transformation rules. Note $|101rangle = |10rangle otimes |1rangle$, and $(Uotimes V)(|xrangle otimes |yrangle) = U|xrangle otimes V|yrangle$, where in our cases (for example) $U = |00ranglelangle00|$, $V = mathbbI_2$, $x=|10rangle$ and $y=|1rangle$:



        $|00ranglelangle00|10rangle otimes mathbbI_2 |1rangle + |01ranglelangle01|10rangle otimes X_2 |1rangle + |10ranglelangle10|10rangle otimes X_2 |1rangle + |11ranglelangle11|10rangle otimes mathbbI_2 |1rangle$



        Now, note we have the following four terms:



        $langle00|10rangle, langle01|10rangle, langle10|10rangle, langle11|10rangle$



        These are called inner products, or dot products and here all of them are zero except for $langle10|10rangle$ - the dot product of $|10rangle$ with itself:



        $langle10|10rangle = beginbmatrix 0 & 0 & 1 & 0endbmatrix
        beginbmatrix0 \ 0 \ 1 \ 0endbmatrix = 1$



        Since the other terms are all zero, they all cancel out:



        $requirecancel cancel + cancel1rangle + |10ranglecdot 1 otimes X_2 |1rangle + cancel1rangle$



        So we are left with:



        $|10rangle otimes X_2 |1rangle$



        Where of course $X_2|1rangle = |0rangle$, so:



        $|10rangle otimes |0rangle = |100rangle$



        And we calculated $U|101rangle = |100rangle$ as expected, without once having to write out a huge inconvenient matrix!






        share|improve this answer
























          up vote
          2
          down vote










          up vote
          2
          down vote









          All quantum operators must be unitary. Unitary means the conjugate-transpose of the operator is its inverse. In your case:



          $UU^dagger = beginbmatrix
          1 & 0 & 0 & 0\
          0 & 1 & 1 & 0\
          0 & 0 & 0 & 0\
          0 & 0 & 0 & 0
          endbmatrix
          beginbmatrix
          1 & 0 & 0 & 0\
          0 & 1 & 0 & 0\
          0 & 1 & 0 & 0\
          0 & 0 & 0 & 0
          endbmatrix =
          beginbmatrix
          1 & 0 & 0 & 0\
          0 & 2 & 0 & 0\
          0 & 0 & 0 & 0\
          0 & 0 & 0 & 0
          endbmatrix$



          So it is most certainly not unitary because $UU^dagger neq mathbbI_4$ (same as your second attempt).



          There's a long way to construct functions like this, and a short way. The long way is to write out all inputs and outputs:



          $U|000rangle = |000rangle$



          $U|001rangle = |001rangle$



          $U|010rangle = |011rangle$



          $U|011rangle = |010rangle$



          $U|100rangle = |101rangle$



          $U|101rangle = |100rangle$



          $U|110rangle = |110rangle$



          $U|111rangle = |111rangle$



          You can then pretty easily construct the operator from this.



          An easier way is to use projection operators & matrix addition to implement "if-then" semantics with matrices:



          $U = |00ranglelangle00| otimes mathbbI_2 + |01ranglelangle01|otimes X_2 + |10ranglelangle10| otimes X_2 + |11ranglelangle11|otimes mathbbI_2$



          The way to read this is "if the input is $|00rangle$ or $|11rangle$, do not flip the third bit. If the input is $|01rangle$ or $|10rangle$, flip the third bit. $|phiranglelangle phi|$ is called the outer product, and is defined for example as follows:



          $|0ranglelangle0| = beginbmatrix 1 \ 0 endbmatrix beginbmatrix 1 & 0 endbmatrix = beginbmatrix 1 & 0 \ 0 & 0 endbmatrix$



          which is called a projection operator. Use projection operators to only apply an operation on specific states - here, $|0rangle$.



          A huge benefit of this projectors & addition approach is that you never have to actually write out the full matrix, which can become enormous as the number of qbits increase - 3-qbit 8x8 matrices already have 64 elements! This is your first step into using symbolic rather than matrix reasoning. For example, we can use the rules of linear algebra to calculate the action of our $U$ on some input:



          $U|101rangle = (|00ranglelangle00| otimes mathbbI_2 + |01ranglelangle01|otimes X_2 + |10ranglelangle10| otimes X_2 + |11ranglelangle11|otimes mathbbI_2)|101rangle$



          Now, matrix multiplication distributes over addition. This means we have:



          $|00ranglelangle00| otimes mathbbI_2 |101rangle + |01ranglelangle01|otimes X_2 |101rangle + |10ranglelangle10| otimes X_2 |101rangle + |11ranglelangle11|otimes mathbbI_2 |101rangle$



          Let's apply further transformation rules. Note $|101rangle = |10rangle otimes |1rangle$, and $(Uotimes V)(|xrangle otimes |yrangle) = U|xrangle otimes V|yrangle$, where in our cases (for example) $U = |00ranglelangle00|$, $V = mathbbI_2$, $x=|10rangle$ and $y=|1rangle$:



          $|00ranglelangle00|10rangle otimes mathbbI_2 |1rangle + |01ranglelangle01|10rangle otimes X_2 |1rangle + |10ranglelangle10|10rangle otimes X_2 |1rangle + |11ranglelangle11|10rangle otimes mathbbI_2 |1rangle$



          Now, note we have the following four terms:



          $langle00|10rangle, langle01|10rangle, langle10|10rangle, langle11|10rangle$



          These are called inner products, or dot products and here all of them are zero except for $langle10|10rangle$ - the dot product of $|10rangle$ with itself:



          $langle10|10rangle = beginbmatrix 0 & 0 & 1 & 0endbmatrix
          beginbmatrix0 \ 0 \ 1 \ 0endbmatrix = 1$



          Since the other terms are all zero, they all cancel out:



          $requirecancel cancel + cancel1rangle + |10ranglecdot 1 otimes X_2 |1rangle + cancel1rangle$



          So we are left with:



          $|10rangle otimes X_2 |1rangle$



          Where of course $X_2|1rangle = |0rangle$, so:



          $|10rangle otimes |0rangle = |100rangle$



          And we calculated $U|101rangle = |100rangle$ as expected, without once having to write out a huge inconvenient matrix!






          share|improve this answer














          All quantum operators must be unitary. Unitary means the conjugate-transpose of the operator is its inverse. In your case:



          $UU^dagger = beginbmatrix
          1 & 0 & 0 & 0\
          0 & 1 & 1 & 0\
          0 & 0 & 0 & 0\
          0 & 0 & 0 & 0
          endbmatrix
          beginbmatrix
          1 & 0 & 0 & 0\
          0 & 1 & 0 & 0\
          0 & 1 & 0 & 0\
          0 & 0 & 0 & 0
          endbmatrix =
          beginbmatrix
          1 & 0 & 0 & 0\
          0 & 2 & 0 & 0\
          0 & 0 & 0 & 0\
          0 & 0 & 0 & 0
          endbmatrix$



          So it is most certainly not unitary because $UU^dagger neq mathbbI_4$ (same as your second attempt).



          There's a long way to construct functions like this, and a short way. The long way is to write out all inputs and outputs:



          $U|000rangle = |000rangle$



          $U|001rangle = |001rangle$



          $U|010rangle = |011rangle$



          $U|011rangle = |010rangle$



          $U|100rangle = |101rangle$



          $U|101rangle = |100rangle$



          $U|110rangle = |110rangle$



          $U|111rangle = |111rangle$



          You can then pretty easily construct the operator from this.



          An easier way is to use projection operators & matrix addition to implement "if-then" semantics with matrices:



          $U = |00ranglelangle00| otimes mathbbI_2 + |01ranglelangle01|otimes X_2 + |10ranglelangle10| otimes X_2 + |11ranglelangle11|otimes mathbbI_2$



          The way to read this is "if the input is $|00rangle$ or $|11rangle$, do not flip the third bit. If the input is $|01rangle$ or $|10rangle$, flip the third bit. $|phiranglelangle phi|$ is called the outer product, and is defined for example as follows:



          $|0ranglelangle0| = beginbmatrix 1 \ 0 endbmatrix beginbmatrix 1 & 0 endbmatrix = beginbmatrix 1 & 0 \ 0 & 0 endbmatrix$



          which is called a projection operator. Use projection operators to only apply an operation on specific states - here, $|0rangle$.



          A huge benefit of this projectors & addition approach is that you never have to actually write out the full matrix, which can become enormous as the number of qbits increase - 3-qbit 8x8 matrices already have 64 elements! This is your first step into using symbolic rather than matrix reasoning. For example, we can use the rules of linear algebra to calculate the action of our $U$ on some input:



          $U|101rangle = (|00ranglelangle00| otimes mathbbI_2 + |01ranglelangle01|otimes X_2 + |10ranglelangle10| otimes X_2 + |11ranglelangle11|otimes mathbbI_2)|101rangle$



          Now, matrix multiplication distributes over addition. This means we have:



          $|00ranglelangle00| otimes mathbbI_2 |101rangle + |01ranglelangle01|otimes X_2 |101rangle + |10ranglelangle10| otimes X_2 |101rangle + |11ranglelangle11|otimes mathbbI_2 |101rangle$



          Let's apply further transformation rules. Note $|101rangle = |10rangle otimes |1rangle$, and $(Uotimes V)(|xrangle otimes |yrangle) = U|xrangle otimes V|yrangle$, where in our cases (for example) $U = |00ranglelangle00|$, $V = mathbbI_2$, $x=|10rangle$ and $y=|1rangle$:



          $|00ranglelangle00|10rangle otimes mathbbI_2 |1rangle + |01ranglelangle01|10rangle otimes X_2 |1rangle + |10ranglelangle10|10rangle otimes X_2 |1rangle + |11ranglelangle11|10rangle otimes mathbbI_2 |1rangle$



          Now, note we have the following four terms:



          $langle00|10rangle, langle01|10rangle, langle10|10rangle, langle11|10rangle$



          These are called inner products, or dot products and here all of them are zero except for $langle10|10rangle$ - the dot product of $|10rangle$ with itself:



          $langle10|10rangle = beginbmatrix 0 & 0 & 1 & 0endbmatrix
          beginbmatrix0 \ 0 \ 1 \ 0endbmatrix = 1$



          Since the other terms are all zero, they all cancel out:



          $requirecancel cancel + cancel1rangle + |10ranglecdot 1 otimes X_2 |1rangle + cancel1rangle$



          So we are left with:



          $|10rangle otimes X_2 |1rangle$



          Where of course $X_2|1rangle = |0rangle$, so:



          $|10rangle otimes |0rangle = |100rangle$



          And we calculated $U|101rangle = |100rangle$ as expected, without once having to write out a huge inconvenient matrix!







          share|improve this answer














          share|improve this answer



          share|improve this answer








          edited 7 hours ago

























          answered 8 hours ago









          ahelwer

          7209




          7209




















              up vote
              0
              down vote













              Your bit strings x in the case when you specify 2 bits are 00, 01, 10, 11.
              Now you want to output the result in another bit/qubit.



              Whether the outpit bit/qubit is initialized as 0 or 1 should not change your unitary operation. The only thing changing is the initial quantum state your system is onto where you apply the unitary operation representing your function.



              In this case, the implementation is straightforward. This is a first CNOT with first qubit of register x as control and the output qubit as target followed by another CNOT with the second qubit of x as control. If you input x=00, nothing happens. If x=01 or 10, you apply one CNOT which adds 1 to the target (definition of CNOT). If x=11, the two CNOTs are applied so you add 1 to the target twice and 1 + 1 = 0 when having one output bit.



              To get the unitary, you can just take the unitary matrix of the CNOT, tensor product with the identity matrix (but respect the order when you write). You do this for the first CNOT giving you one unitary $U_1$, then the second giving $ U_2 $ and finally multiply $ U_2 * U_1 $ to get your unitary operator.






              share|improve this answer
























                up vote
                0
                down vote













                Your bit strings x in the case when you specify 2 bits are 00, 01, 10, 11.
                Now you want to output the result in another bit/qubit.



                Whether the outpit bit/qubit is initialized as 0 or 1 should not change your unitary operation. The only thing changing is the initial quantum state your system is onto where you apply the unitary operation representing your function.



                In this case, the implementation is straightforward. This is a first CNOT with first qubit of register x as control and the output qubit as target followed by another CNOT with the second qubit of x as control. If you input x=00, nothing happens. If x=01 or 10, you apply one CNOT which adds 1 to the target (definition of CNOT). If x=11, the two CNOTs are applied so you add 1 to the target twice and 1 + 1 = 0 when having one output bit.



                To get the unitary, you can just take the unitary matrix of the CNOT, tensor product with the identity matrix (but respect the order when you write). You do this for the first CNOT giving you one unitary $U_1$, then the second giving $ U_2 $ and finally multiply $ U_2 * U_1 $ to get your unitary operator.






                share|improve this answer






















                  up vote
                  0
                  down vote










                  up vote
                  0
                  down vote









                  Your bit strings x in the case when you specify 2 bits are 00, 01, 10, 11.
                  Now you want to output the result in another bit/qubit.



                  Whether the outpit bit/qubit is initialized as 0 or 1 should not change your unitary operation. The only thing changing is the initial quantum state your system is onto where you apply the unitary operation representing your function.



                  In this case, the implementation is straightforward. This is a first CNOT with first qubit of register x as control and the output qubit as target followed by another CNOT with the second qubit of x as control. If you input x=00, nothing happens. If x=01 or 10, you apply one CNOT which adds 1 to the target (definition of CNOT). If x=11, the two CNOTs are applied so you add 1 to the target twice and 1 + 1 = 0 when having one output bit.



                  To get the unitary, you can just take the unitary matrix of the CNOT, tensor product with the identity matrix (but respect the order when you write). You do this for the first CNOT giving you one unitary $U_1$, then the second giving $ U_2 $ and finally multiply $ U_2 * U_1 $ to get your unitary operator.






                  share|improve this answer












                  Your bit strings x in the case when you specify 2 bits are 00, 01, 10, 11.
                  Now you want to output the result in another bit/qubit.



                  Whether the outpit bit/qubit is initialized as 0 or 1 should not change your unitary operation. The only thing changing is the initial quantum state your system is onto where you apply the unitary operation representing your function.



                  In this case, the implementation is straightforward. This is a first CNOT with first qubit of register x as control and the output qubit as target followed by another CNOT with the second qubit of x as control. If you input x=00, nothing happens. If x=01 or 10, you apply one CNOT which adds 1 to the target (definition of CNOT). If x=11, the two CNOTs are applied so you add 1 to the target twice and 1 + 1 = 0 when having one output bit.



                  To get the unitary, you can just take the unitary matrix of the CNOT, tensor product with the identity matrix (but respect the order when you write). You do this for the first CNOT giving you one unitary $U_1$, then the second giving $ U_2 $ and finally multiply $ U_2 * U_1 $ to get your unitary operator.







                  share|improve this answer












                  share|improve this answer



                  share|improve this answer










                  answered 4 hours ago









                  cnada

                  96818




                  96818



























                       

                      draft saved


                      draft discarded















































                       


                      draft saved


                      draft discarded














                      StackExchange.ready(
                      function ()
                      StackExchange.openid.initPostLogin('.new-post-login', 'https%3a%2f%2fquantumcomputing.stackexchange.com%2fquestions%2f4442%2fwhat-is-the-matrix-for-the-operator-that-implements-a-function-to-tell-the-parit%23new-answer', 'question_page');

                      );

                      Post as a guest













































































                      Comments

                      Popular posts from this blog

                      Long meetings (6-7 hours a day): Being “babysat” by supervisor

                      Is the Concept of Multiple Fantasy Races Scientifically Flawed? [closed]

                      Confectionery